A ball is thrown straight up with an initial velocity of 35.7 m/s. How long does
it take to hit the ground?

Answers

Answer 1

y = v₀t - gt²/2

For y=0 (back to where we started),

0 = t(v₀ - gt/2)

t=0 (which we knew) or v₀ - gt/2 = 0

v₀ = gt/2

2v₀ = gt

t = 2v₀/g

t = 2 (35.7 m/s) / (9.8 m/s/s) = 7.3 seconds

Answer: 7.3 seconds


Related Questions

pls help me I need it I will give u 100 points and mark u brainliest pls help me. I need 3 answers 3,4,5

Answers

Answer:

Question 3. Option B,

Question 4. Option D,

Question 5. Option B

Step-by-step explanation:

Question 3.

If we have a table, we can note that each juice has a proportionality among one another - comparative to the number of servings. If we can figure out this proportionality, we can apply it to the servings of grapefruit juice;

[tex]8 / 16 = 1 / 2,\\4 c / 8c = 1 / 2,\\1 c / 2 c = 1 / 2[/tex]

As we can tell, this " proportion " is 1 to 2. Thus, the amount of grapefruit for 16 servings is 2 * 3 = 6c - Option B

Question 4.

The distance Carlos biked is 24 miles, the total distance he covered is 30 miles;

[tex]24 : 30 = 4 : 5,\\4 to 5,\\4 : 5,\\4 / 5\\\\Conclusion - Option D[/tex]

Question 5.

If 1 / 6 of the fruits are oranges, there must be 6 times of the other fruit. Thus, the solution is Option B

Answer:

Question 3: 6c

Question 4:  4 to 5, 4:5, 4/5

Carlos biked 24 miles and ran 6 miles. The total distance is equal to 30 miles.

The question is: How much of the total distance he biked?

[tex]$\frac{24}{30}=\frac{4}{5} $[/tex]

Question 5: 3 oranges and 15 apples

[tex]$\frac{1}{6} $[/tex]  of the fruits are oranges.

It means that approximately [tex]16.6\%[/tex] of the fruits are oranges. If there were 6 fruits, 1 is orange and 5 are apples, therefore B is wrong.

If you take the options, only the first one is correct.


The following table shows the probability distribution for a discrete random
variable.
Х
11
14
16
19
21
23
24
29
P(X) 0.07 0.21
0.17 0.25 0.05 0.04 0.13 0.08
What is the mean of this discrete random variable? (That is, what is E(X), the
expected value of X?)
A. 19
B. 19.625
C. 20
D. 18.59

Answers

Answer:

Because the random variable X is discrete, the variance of X is defined as

var(X) = 

where

p =  values of probability as given in the table.

μ = 18.59

Calculate the array (x - μ).

x-μ = [-7.59 -4.59 -2.59 0.41 2.41 4.41 5.41 10.41]

Calculate the array (x-μ)².

(x- μ)² = [57.6081 21.0681 6.7081 0.1681 5.8081 19.4481 29.2681 108.3681]

Calculate the array p*(x-μ)².

p*(x- μ)² = [4.0326 4.4342 1.1404 0.042 0.2904 0.7779 3.8049 8.6694]

Calculate the variance of X.

var(X) = 23.182

Step-by-step explanation:

Answer: 23.18 (nearest hundredth)

Answer: 18.59

Step-by-step explanation:

A P E X

A bicycle tire is 28 inches in diameter. Approximately how far does the bicycle move forward
each time the wheel goes around? (Use as an approximation for 7r.)?

Answers

Answer: 88 inches

Step-by-step explanation:

So the radius of the tire is computed by dividing it in half. We know that in one round of the wheel, the wheel moves to a distance equal to the circumference of the wheel. So each time the wheel goes around, the bicycle moves about 88 inches 88 i n c h e s forward.

Answer:

The answer would be=

88 Inches!

Step-by-step explanation:

I hope you do well!

Ram baked 946 cupcakes she sold them in a box of 22 for $16.50 a box how many boxes would she need to make at least $300?

Answers

this app is g a y ksjshfusjwnsbx

100 points, will give brainliest!
Suppose that in circle with center P, and a central angle, ∠OPQ, intersects minor arc OQ, where the measure of arc OQ = 82◦, and the measure of ∠OPQ is (5x – 3) ◦.

What is the value of x?

Thank you so much for helping!

Answers

Hope you are doing well

Answer:

x=17

for the fun of it
1 * 100000000 * 967 *0 +1

Answers

Answer:

1.

Step-by-step explanation:

By the order of operations, multiplication goes before addition. We got left to right.

As the entire resultant expression of the first two operators is multiplied by 0, we are left with 0 + 1.

0 + 1 = 1.

I hope this helps!

Order the expressions from the least to the greatest
-2, -2.2, 5, 2, 2.2, -5

Answers

Answer:

-5, -2.2, -2, 2, 2.2, 5

Step-by-step explanation:

-5

-2.2

-2

2

2.2

5

Answer:

-5

-2.2

-2

2

2.2

5

Select the correct product.
(y+ z)3
y 3+3y2z+ 3yz 2 + z 3
y 3 y+ 3 z
y 3 z 3
y 3 + z 3​

Answers

Answer:

Step-by-step explanation:

Ben makes $39,600 a year. What is the maximum amount he can afford for a'


mortgage each month?


O


O


O


A. $890


B. $825


c. $850


D. $875

Answers

Answer: I feel that the answer might be B. $825 because I did calculate it but I lost my calculations so the answer is B but I just don't have the reason why sorry

The maximum amount Ban can afford for a' mortgage each month when he makes $39,600 a year is $825. Option B is correct.

What is mortgage?

The mortgage is a type of loan which is taken to purchase the home or lands.

In the given problem, Ben makes $39,600 a year. The rate of interset for the mortgage he has is 4. Thus, the amount he can afford for a'mortgage each year is,

[tex]M=\dfrac{39600}{4}\\M=9900[/tex]

There is total 12 months in one year. Thus, the maximum amount he can afford for a'mortgage each month is,

[tex]M=\dfrac{9900}{12}\\M=825[/tex]

Thus, the maximum amount Ban can afford for a' mortgage each month when he makes $39,600 a year is $825. Option B is correct.

Learn more about the mortgage here;

https://brainly.com/question/22598793

#SPJ2

Use Pythagorean’s Theorem

Answers

Answer:

x =3.9

Step-by-step explanation:

Since this is a right triangle we can use the Pythagorean theorem

a^2 + b^2 =c^2

x^2 + (5x)^2 = 20 ^2

x^2 + 25x^2 = 400

Combine like terms

26 x^2 = 400

Divide each side by 26

x^2 = 400/26

Take the square root of each side

sqrt(x^2) = sqrt(400/26)

x = sqrt(400/26)

x =3.922322703

Rounding to the nearest tenth

x =3.9

Answer:

3.9

Step-by-step explanation:

:)

answer please need help and is struggling

Answers

Answer:

q < 5

Step-by-step explanation:

Since there is an open circle it means there is no equal sign

The line goes to the left so it is less then q < 5

Answer:

q<5

Step-by-step explanation:

The sign is < because the arrow is point left and has an open circle. 5 because that is where the arrow starts.

Somebody please help me please

Answers

Answer:

The correct sign is : <

Step-by-step explanation:

3.126 *10^2 is  greater than 31260000*10^-9 by  312.56874

Answer: <

Step-by-step explanation:

10^-9 = .00000001

Multiply

31,260,000 * .00000001 = .3126 * 10^0 (1)

10^2 = 100

Multiply

3.126 * 100 = 312.6

.3126 * 10^0 < 3.126 * 10^2

Solve the equation. Then check your solution.
-
(3a + 4) = 7
Helppp

Answers

Answer:

1

Step-by-step explanation:

-3a+4 =7

-4. _4

-3a=3

divide by -3 on both sides

a=-1

3+4=7

Answer:

a= [tex]-3\frac{2}{3}[/tex] or ≈ -3.66

Step-by-step explanation:

-(3a+4)=7

-3a-4=7

Add 4 to both sides

-3a=11

Multiply both sides by -1

3a= -11

Divide both sides by 3

a= [tex]-3\frac{2}{3}[/tex]

answer options : 120, 60, 180 or 90

Answers

180 I believe - sorry if it’s wrong :(

Find the equation of the line.
Use exact numbers
y=___ x+____

Answers

Answer:

y=x-5

Step-by-step explanation:

transformations from parent function:

down 5 units

Answer:

y=x-5

Step-by-step explanation:

slope=rise/run=5/5=1

y -intercept=-5

y=x-5

Select the ordered pair that is a solution
of this equation.
2x - 4y = 2
Click on the correct answer.
(-2,-3)
(-3,-2)
(-3,-6)​

Answers

Step-by-step explanation:

You could do this in your head, but I'm going to type out everything.

Also, you're going to have to just test these out, there is no short cut for this sort of problem.

[tex]2(-2)-4(-3)=2\\-4+12=2\\8\neq 2\\\\2(-3)-4(-2)=2\\-6+8=2\\2=2[/tex]

I didn't do the last ordered pair because we already found one that fit the equation. Also, it wouldn't be a 1:1 equation if that last one was true, so you can rule it out once you see that (-3,-2) is the correct answer.

Answer:

(-3,-2)

Find the lowest common denominator

Answers

Answer:

A #1 ( x+2)2 (x-2)2

Step-by-step explanation:

it is the only one that could possibly work

Name the types of angles shown check all that apply

Answers

There are no other options listed

Answer:

There are straight, zero, right, obtuse,acute, complementary, supplementary, vertical

Step-by-step explanation:

These are a few I know of

What is the equation of the graphed line written in standard form?


A. x=-3

B. y=-3

C. x+y=-3

D. x-y=-3
I know the pictures black It's just showing a vertical line going through -3 on the x-axis (The blue line)

Answers

The answer should be A

Which of the following is an offer? Select one: a. Hedva walks into the Ferrari Dealership and says, “I am going to buy a car.” b. Lucy says to Zehmer “I bet you wouldn’t sell this house for $50,000.” c. "This car isn't worth what you're charging. d. Bruo says, “I will pay $25 for this book.”

Answers

Answer:

D, "I will pay $25 for this book."

Step-by-step explanation

It would be d, that is correct

If f(x) = 2x - 3 and g(x) = x^2 + 3, what is f(x) - g(x)?

Answers

(2x - 3) - (x^2 + 3)

2x - 3 - x^2 + 3 Remove parentheses...

2x - 3 - x^2 - 3 Collect like terms and solve...

2x - 6 - x^2 Re-order terms...


ANSWER: -x^2 + 2x - 6



Answer:

the answer is  -x^2 + 2x - 6

Step-by-step explanation:

(8^3 + 2^2 + 1) - (7 - 7^3+3^2)

Answers

Answer:

844 is the ans

Step-by-step explanation:

512+4+1 -7 +343+9= 844

Answer: 844

Step-by-step explanation:

What is... 55.
One more?.......,,,
One less?.............
Ten more?...........,
Ten less?..............

Answers

Answer:

one more would be 56

one less would be 54

ten more would 65

ten less would be 45

Step-by-step explanation:

add one more to 55 to get 56

subtract one from 55 to get 54

add ten more to 55 to get 65

subtract ten from 55 to get 45

20 POINTS!! Let logb A = 3; log, C = 2; log, D= 5
What is the value of logb(a5c2/d6)?

Answers

Answer:

-11

Step-by-step explanation:

logb ( a^5 c^2 / d^6)

Splitting these into log (xy/z) = log x + log y  -  logz

logb ( a^5)  +logb ( c^2)  - logb( d^6)

We also know that log x^y = y log x

5logb ( a)  +2logb ( c)  - 6 logb( d)

Given logb ( a)=3  logb ( c) =2   logb( d)=5

5 * 3 + 2* 2 - 6* 5

15 +4 -30

19 - 30

-11

Answer: -11

Step-by-step explanation:

just got it right on ap3x

The net of a prism is shown.
4 ft
5 ft
4 ft
5 ft
6 ft
3 ft 1
4 ft
5 ft
What is the surface area of the prism?

Answers

Answer:

Step-by-step explanation:

surface area=4×5+2×1/2×3×4+5×5+3×6

=20+12+25+18

=75

Mrs. Holland purchased a computer. She initially put down $105.40 and will pay $20.60 per
month until the computer is paid off. Use the equation b = 105.40 + 20.60m to determine
how many months it will take to pay off the computer if the total cost was $682.20

Answers

Answer:

It will take her 28 months

Step-by-step explanation:

You first subtract 682.20 and 150.40 to get 576.80. Then you divide 576.80 by 20.60 to get 28.

Answer:

The Answer you are looking for is 28 months.

I need help again brainiest who ever get this right

Answers

Answer:

Step-by-step explanation:

26x25x24x23x22x21x10x9=30,891,577,600

=Thirty billion, eight hundred ninety-one million, five hundred seventy-seven thousand, six hundred.

the answer was kind of in the question btw :)

The volume of a cylinder is 441 cubic centimeters and its radius is 7 centimeters.
What is the height of the cylinder?
Enter your answer in the box.

Answers

Answer:

689 ft

Step-by-step explanation:

Answer:

2.86

Step-by-step explanation:

h=V

πr2=441

π·72≈2.86479

A cylindrical tin of height 15 cm and radius 4 cm is filled with sand from a rectangular box. How many times can the tin be filled if the dimensions of the box are 50 cm by 40 cm by 20 cm?

Answers

Answer:

Volume of Box / volume of tin = number of tins required

Volume of box =50*40*20= 40000 cu.cm

Volume of Tin = pi*r^2*h=pi*4^2*15 = 754 cu.cm

4000/754= 53 tins (rounded off)

Step-by-step explanation: i'm sorry if this is a bit complicated but i hope it answers your question. Take care.

Give an example of an absolute value inequality whose answers falls between two values. Give an example of an absolute value inequality whose answers fall outside of two values. What creates the difference? Be specific. *

Answers

Answer:

A) Suppose we have an onedimensional situation.

in the 0 of our x-axis, we have a fruit tree, and we want to rest at a distance no bigger than 6 ft of the tree, then all the possible positions of our resting place are:

x ∈(-6ft, 6ft)

we can write this as: IxI < 6ft

b) now we think the opposite situation, we want to rest at least 6ft away from the tree, then we have that:

x ∉  [-6ft, 6ft].

or IxI > 6ft.

So you can see that the difference in those two cases is if we want to be "inside a given range" (for the first case) or "outside a given range" (for the second case).

Other Questions
find the total surface area of this cylinder. Give yor answer to 1 decimal place. Award is brainliest answer Graph the line with slope -1/3 and y -intercept 6 . brainstorming involves? can you please solve this! How might you create interest in a design in which all the shapes are organic? to drive, conduct in spanish A science club has 16 members. How many ways can a president, a Vice President, and a treasurer be selected from the members? Jordan invested $8,500 in an account paying an interest rate of 4.1% compounded daily. Assuming no deposits or withdrawals are made, how long would it take, to the nearest year, for the value of the account to reach $13,070? ii.State the sentence pattern of the following sentences.1. I like football.2. They elected him president.3. I swim.4. We met outside.5. Kofi is a boy. The Strange Case of Dr. Jekyll and Mr. Hyde (1886)Which sentence creates a mood of foreboding in this selection Show that T is a linear transformation by finding a matrix that implements the mapping. Note that x1, x2, ... are not vectors but are entries in vectors. T(X1,X2,X3,X4) = (x1 +4x2, 0, 3x2 +x4, x2 -x4) 8+8? Pls help will mark you or whatever Solve the inequality 5(2x + 1) < 10.A. x < 1/2B. x > 1/2C. x < 3/2D. x > 3/2 What Editing software can i download on Computer to edit my videos? M5-3/2x less than or equal to 1/3 Find the value of y. -6y+14+4y=32 What would be the approximate 95% confidence interval for the mean number of ounces of catchup bottle in the sample Bonsoir, quelquun pourrais me aider pour quelques exercices en grammaire, sil vous plat?(Je vous met les exercise en pice jointe) Merci beaucoup!:) Solve for x in the diagram below.1203x02= Which of the following is an angle shown in the drawing?Will mark as brainliest if correct. Please answer ASAP please.